Which one of the following must be false?

Ecochran on October 7, 2020

Deductions and #21

Hi, just like the other two folks here, would love a response. Very not assuring that someone asked a question three months ago, never got a response, and we are told if there aren't video discussions to rely on the message boards. Well lots of times there are neither when no one responds, so....?

Reply
Create a free account to read and take part in forum discussions.

Already have an account? log in

Emil-Kunkin on April 20 at 08:14PM

For a must be false we are trying to break this game. Here, the two easiest ways would be to break apart JG, or to force GK to be far apart.

I would first look for something that breaks up our block of JG, and the easiest way to break that would be to put one thing in the middle day that is neither j nor g. Thankfully, we have an answer choice that does just that in the form of c